Partial Simplified Proof for the prime version of the Catalan ConjectureAttempting a Proof of the Prime Version of Catalan's Conjecture$p$ an odd prime, $p equiv 3 pmod 8$. Show that $2^(fracp-12)*(p-1)! equiv 1 pmod p$direct proof $ x^2 pm 1$ is not a perfect cube for integer $ xgeq 4$Proof concerning specific class of Proth numbersUsing CRT to reason about powers of $2$ where the power is two less than an odd primeSimplified variant of Collatz conjecture.Simplifying the proof behind Catalan's ConjectureAttempting a Proof of the Prime Version of Catalan's ConjectureIf $a$ and $-a$ are both quadratic residues mod an odd prime $p$, then $p equiv 1pmod4$On the conjecture that $12mid t_n + t_n+1$.Proving for $a>1,b > 1$, $2^a ne p^b + 1$

Stopping power of mountain vs road bike

What exploit are these user agents trying to use?

Did converts (ger tzedek) in ancient Israel own land?

Is it possible to create light that imparts a greater proportion of its energy as momentum rather than heat?

How do conventional missiles fly?

Will google still index a page if I use a $_SESSION variable?

1960's book about a plague that kills all white people

Can a virus destroy the BIOS of a modern computer?

I'm flying to France today and my passport expires in less than 2 months

UK: Is there precedent for the governments e-petition site changing the direction of a government decision?

Why can't we play rap on piano?

Why is consensus so controversial in Britain?

Assassin's bullet with mercury

Alternative to sending password over mail?

How much of data wrangling is a data scientist's job?

Can one be a co-translator of a book, if he does not know the language that the book is translated into?

How to show the equivalence between the regularized regression and their constraint formulas using KKT

In a Spin are Both Wings Stalled?

Took a trip to a parallel universe, need help deciphering

Doing something right before you need it - expression for this?

What is going on with Captain Marvel's blood colour?

Emailing HOD to enhance faculty application

Why do I get two different answers for this counting problem?

What is the word for reserving something for yourself before others do?



Partial Simplified Proof for the prime version of the Catalan Conjecture


Attempting a Proof of the Prime Version of Catalan's Conjecture$p$ an odd prime, $p equiv 3 pmod 8$. Show that $2^(fracp-12)*(p-1)! equiv 1 pmod p$direct proof $ x^2 pm 1$ is not a perfect cube for integer $ xgeq 4$Proof concerning specific class of Proth numbersUsing CRT to reason about powers of $2$ where the power is two less than an odd primeSimplified variant of Collatz conjecture.Simplifying the proof behind Catalan's ConjectureAttempting a Proof of the Prime Version of Catalan's ConjectureIf $a$ and $-a$ are both quadratic residues mod an odd prime $p$, then $p equiv 1pmod4$On the conjecture that $12mid t_n + t_n+1$.Proving for $a>1,b > 1$, $2^a ne p^b + 1$













5












$begingroup$


I have found an elementary approach which seems to work in proving that Catalan's conjecture is true for specific primes. I am attempting to generalize this approach to see how far it will go.



I am presenting it here with the hopes of finding a mistake or confirming that this approach is worth exploring further.



From Wikipedia, the Catalan Conjecture can be defined as:




  • Let $a>1,b>1,x>0,y>0$


  • Then if:


$x^a - y^b = 1$



  • The only solution is $x=3, a=2, y=2, b=3$



I will attempt now to prove the case where:




$x=2, y$ is an odd prime




(1) Making this assumption, we have:




$$2^a = 2left[y(y^b-2 + y^b-3 + dots + 1)(fracy-12) + fracy+12right]$$




(2) $fracy+12$ is even




If $fracy+12$ is odd, then $fracy-12$ is even and $left[y(y^b-2+dots+1)(fracy-12) + fracy+12right]$ is odd.




(3) $b$ is odd




If $fracy+12$ is even, then $fracy-12$ is odd and it follows that $(y^b-2 + dots + 1)$ must be even. Therefore, $b-2$ must be odd.




(4) Let $2^u$ be the highest power of $2$ that divides $fracy+12$



(5) $y equiv -1 pmod 2^u+1$ since:




There exists $m$ such that $fracy+12^u+1=2m+1$ which means $y = 2^u+1(2m + 1) - 1$




(6) $2^u+1 | (y^b-2 + dots 1)$ since:




$b-2$ is odd and $(y^b-2 + y^b-3 + dots + y + 1) equiv (-1 + 1) +dots + (-1 + 1) equiv 0 pmod 2^u+1$




(7) But then we have a contradiction since:




$y(fracy^b-2+dots+12^u)(fracy-12)$ is even but $fracy+12^u+1$ is odd so that:



$$2^a ne 2^u+1left[y(frac^b-2+dots+12^u)(fracy-12) + fracy+12^u+1right]$$





Edit: Attempting to greatly simplify the argument based on feedback received.










share|cite|improve this question











$endgroup$
















    5












    $begingroup$


    I have found an elementary approach which seems to work in proving that Catalan's conjecture is true for specific primes. I am attempting to generalize this approach to see how far it will go.



    I am presenting it here with the hopes of finding a mistake or confirming that this approach is worth exploring further.



    From Wikipedia, the Catalan Conjecture can be defined as:




    • Let $a>1,b>1,x>0,y>0$


    • Then if:


    $x^a - y^b = 1$



    • The only solution is $x=3, a=2, y=2, b=3$



    I will attempt now to prove the case where:




    $x=2, y$ is an odd prime




    (1) Making this assumption, we have:




    $$2^a = 2left[y(y^b-2 + y^b-3 + dots + 1)(fracy-12) + fracy+12right]$$




    (2) $fracy+12$ is even




    If $fracy+12$ is odd, then $fracy-12$ is even and $left[y(y^b-2+dots+1)(fracy-12) + fracy+12right]$ is odd.




    (3) $b$ is odd




    If $fracy+12$ is even, then $fracy-12$ is odd and it follows that $(y^b-2 + dots + 1)$ must be even. Therefore, $b-2$ must be odd.




    (4) Let $2^u$ be the highest power of $2$ that divides $fracy+12$



    (5) $y equiv -1 pmod 2^u+1$ since:




    There exists $m$ such that $fracy+12^u+1=2m+1$ which means $y = 2^u+1(2m + 1) - 1$




    (6) $2^u+1 | (y^b-2 + dots 1)$ since:




    $b-2$ is odd and $(y^b-2 + y^b-3 + dots + y + 1) equiv (-1 + 1) +dots + (-1 + 1) equiv 0 pmod 2^u+1$




    (7) But then we have a contradiction since:




    $y(fracy^b-2+dots+12^u)(fracy-12)$ is even but $fracy+12^u+1$ is odd so that:



    $$2^a ne 2^u+1left[y(frac^b-2+dots+12^u)(fracy-12) + fracy+12^u+1right]$$





    Edit: Attempting to greatly simplify the argument based on feedback received.










    share|cite|improve this question











    $endgroup$














      5












      5








      5


      3



      $begingroup$


      I have found an elementary approach which seems to work in proving that Catalan's conjecture is true for specific primes. I am attempting to generalize this approach to see how far it will go.



      I am presenting it here with the hopes of finding a mistake or confirming that this approach is worth exploring further.



      From Wikipedia, the Catalan Conjecture can be defined as:




      • Let $a>1,b>1,x>0,y>0$


      • Then if:


      $x^a - y^b = 1$



      • The only solution is $x=3, a=2, y=2, b=3$



      I will attempt now to prove the case where:




      $x=2, y$ is an odd prime




      (1) Making this assumption, we have:




      $$2^a = 2left[y(y^b-2 + y^b-3 + dots + 1)(fracy-12) + fracy+12right]$$




      (2) $fracy+12$ is even




      If $fracy+12$ is odd, then $fracy-12$ is even and $left[y(y^b-2+dots+1)(fracy-12) + fracy+12right]$ is odd.




      (3) $b$ is odd




      If $fracy+12$ is even, then $fracy-12$ is odd and it follows that $(y^b-2 + dots + 1)$ must be even. Therefore, $b-2$ must be odd.




      (4) Let $2^u$ be the highest power of $2$ that divides $fracy+12$



      (5) $y equiv -1 pmod 2^u+1$ since:




      There exists $m$ such that $fracy+12^u+1=2m+1$ which means $y = 2^u+1(2m + 1) - 1$




      (6) $2^u+1 | (y^b-2 + dots 1)$ since:




      $b-2$ is odd and $(y^b-2 + y^b-3 + dots + y + 1) equiv (-1 + 1) +dots + (-1 + 1) equiv 0 pmod 2^u+1$




      (7) But then we have a contradiction since:




      $y(fracy^b-2+dots+12^u)(fracy-12)$ is even but $fracy+12^u+1$ is odd so that:



      $$2^a ne 2^u+1left[y(frac^b-2+dots+12^u)(fracy-12) + fracy+12^u+1right]$$





      Edit: Attempting to greatly simplify the argument based on feedback received.










      share|cite|improve this question











      $endgroup$




      I have found an elementary approach which seems to work in proving that Catalan's conjecture is true for specific primes. I am attempting to generalize this approach to see how far it will go.



      I am presenting it here with the hopes of finding a mistake or confirming that this approach is worth exploring further.



      From Wikipedia, the Catalan Conjecture can be defined as:




      • Let $a>1,b>1,x>0,y>0$


      • Then if:


      $x^a - y^b = 1$



      • The only solution is $x=3, a=2, y=2, b=3$



      I will attempt now to prove the case where:




      $x=2, y$ is an odd prime




      (1) Making this assumption, we have:




      $$2^a = 2left[y(y^b-2 + y^b-3 + dots + 1)(fracy-12) + fracy+12right]$$




      (2) $fracy+12$ is even




      If $fracy+12$ is odd, then $fracy-12$ is even and $left[y(y^b-2+dots+1)(fracy-12) + fracy+12right]$ is odd.




      (3) $b$ is odd




      If $fracy+12$ is even, then $fracy-12$ is odd and it follows that $(y^b-2 + dots + 1)$ must be even. Therefore, $b-2$ must be odd.




      (4) Let $2^u$ be the highest power of $2$ that divides $fracy+12$



      (5) $y equiv -1 pmod 2^u+1$ since:




      There exists $m$ such that $fracy+12^u+1=2m+1$ which means $y = 2^u+1(2m + 1) - 1$




      (6) $2^u+1 | (y^b-2 + dots 1)$ since:




      $b-2$ is odd and $(y^b-2 + y^b-3 + dots + y + 1) equiv (-1 + 1) +dots + (-1 + 1) equiv 0 pmod 2^u+1$




      (7) But then we have a contradiction since:




      $y(fracy^b-2+dots+12^u)(fracy-12)$ is even but $fracy+12^u+1$ is odd so that:



      $$2^a ne 2^u+1left[y(frac^b-2+dots+12^u)(fracy-12) + fracy+12^u+1right]$$





      Edit: Attempting to greatly simplify the argument based on feedback received.







      elementary-number-theory proof-verification prime-numbers modular-arithmetic






      share|cite|improve this question















      share|cite|improve this question













      share|cite|improve this question




      share|cite|improve this question








      edited Feb 22 '17 at 22:55







      Larry Freeman

















      asked Feb 20 '17 at 23:52









      Larry FreemanLarry Freeman

      3,27921240




      3,27921240




















          1 Answer
          1






          active

          oldest

          votes


















          1












          $begingroup$

          I couldn't find any errors so your approach seems to be correct.



          Note, though, that your solution is more general in that it also works for any $x$ being a positive power of $2$, plus for all odd $y gt 1$, not just odd primes. For the first case, this is because your proof just requires the LHS to be a power of $2$. For the second case, this is due to you only using that $y$ is odd, and in particular not using any particular property of primes, in your solution.



          As for "exploring further" with this approach, I'm not quite sure what you mean. However, note you can use a similar approach to show that the only values which work for $y$ being a positive power of $2$ is the only known solution. To show this, you can follow a similar approach to what you did. First, let $y = 2^c$ for an integer $c ge 1$. Next, the equation can now be rewritten as



          $$2^bc = x^a - 1 tag1labeleq1$$



          This shows that $x$ must be odd. First, consider that $a$ is even, i.e., $a = 2d$. Then $x^a - 1 = left(x^d + 1right)left(x^d - 1right)$, so both $x^d + 1$ and $x^d - 1$ must be powers of $2$. First, $x^d - 1 = 1 ; Rightarrow ; x^d = 2$, so it's not possible. Next, $x^d - 1 = 2 ; Rightarrow ; x^d = 3$, so $x = 3$ and $d = 1$, giving the $1$ known solution. No larger values of $x^d - 1$ as a power of $2$ have that $x^d + 1$ is also a power of $2$.



          Next, consider that $a$ is odd. Factoring the LHS gives $x^a - 1 = left(x - 1right)left(x^a-1 + x^a-2 + cdots + x + 1right)$. Since $x$ is odd and there are $a$ terms in the second factor, this factor must be odd and $gt 1$. However, the LHS of eqrefeq1 is a power of $2$, so it has no odd factors. Thus, $a$ can't be an odd integer.



          Putting this solution together with what you've shown proves that, apart from the one known solution, there are no other ones where $x$ or $y$ is a power of $2$.



          You could try to do something similar with odd prime numbers, but I don't believe that just these sorts of basic remainder and number of factors type arguments are sufficient to show there is no solution if either $x$ or $y$ is of a given simple form (e.g., just a prime or a power of a prime). If you haven't already done so, you may wish to read & analyze Preda Mihăilescu's proof, such as discussed & outlined in the AMS article at Catalan's Conjecture: Another Old Diophantine Problem Solved.






          share|cite|improve this answer









          $endgroup$













            Your Answer





            StackExchange.ifUsing("editor", function ()
            return StackExchange.using("mathjaxEditing", function ()
            StackExchange.MarkdownEditor.creationCallbacks.add(function (editor, postfix)
            StackExchange.mathjaxEditing.prepareWmdForMathJax(editor, postfix, [["$", "$"], ["\\(","\\)"]]);
            );
            );
            , "mathjax-editing");

            StackExchange.ready(function()
            var channelOptions =
            tags: "".split(" "),
            id: "69"
            ;
            initTagRenderer("".split(" "), "".split(" "), channelOptions);

            StackExchange.using("externalEditor", function()
            // Have to fire editor after snippets, if snippets enabled
            if (StackExchange.settings.snippets.snippetsEnabled)
            StackExchange.using("snippets", function()
            createEditor();
            );

            else
            createEditor();

            );

            function createEditor()
            StackExchange.prepareEditor(
            heartbeatType: 'answer',
            autoActivateHeartbeat: false,
            convertImagesToLinks: true,
            noModals: true,
            showLowRepImageUploadWarning: true,
            reputationToPostImages: 10,
            bindNavPrevention: true,
            postfix: "",
            imageUploader:
            brandingHtml: "Powered by u003ca class="icon-imgur-white" href="https://imgur.com/"u003eu003c/au003e",
            contentPolicyHtml: "User contributions licensed under u003ca href="https://creativecommons.org/licenses/by-sa/3.0/"u003ecc by-sa 3.0 with attribution requiredu003c/au003e u003ca href="https://stackoverflow.com/legal/content-policy"u003e(content policy)u003c/au003e",
            allowUrls: true
            ,
            noCode: true, onDemand: true,
            discardSelector: ".discard-answer"
            ,immediatelyShowMarkdownHelp:true
            );



            );













            draft saved

            draft discarded


















            StackExchange.ready(
            function ()
            StackExchange.openid.initPostLogin('.new-post-login', 'https%3a%2f%2fmath.stackexchange.com%2fquestions%2f2153827%2fpartial-simplified-proof-for-the-prime-version-of-the-catalan-conjecture%23new-answer', 'question_page');

            );

            Post as a guest















            Required, but never shown

























            1 Answer
            1






            active

            oldest

            votes








            1 Answer
            1






            active

            oldest

            votes









            active

            oldest

            votes






            active

            oldest

            votes









            1












            $begingroup$

            I couldn't find any errors so your approach seems to be correct.



            Note, though, that your solution is more general in that it also works for any $x$ being a positive power of $2$, plus for all odd $y gt 1$, not just odd primes. For the first case, this is because your proof just requires the LHS to be a power of $2$. For the second case, this is due to you only using that $y$ is odd, and in particular not using any particular property of primes, in your solution.



            As for "exploring further" with this approach, I'm not quite sure what you mean. However, note you can use a similar approach to show that the only values which work for $y$ being a positive power of $2$ is the only known solution. To show this, you can follow a similar approach to what you did. First, let $y = 2^c$ for an integer $c ge 1$. Next, the equation can now be rewritten as



            $$2^bc = x^a - 1 tag1labeleq1$$



            This shows that $x$ must be odd. First, consider that $a$ is even, i.e., $a = 2d$. Then $x^a - 1 = left(x^d + 1right)left(x^d - 1right)$, so both $x^d + 1$ and $x^d - 1$ must be powers of $2$. First, $x^d - 1 = 1 ; Rightarrow ; x^d = 2$, so it's not possible. Next, $x^d - 1 = 2 ; Rightarrow ; x^d = 3$, so $x = 3$ and $d = 1$, giving the $1$ known solution. No larger values of $x^d - 1$ as a power of $2$ have that $x^d + 1$ is also a power of $2$.



            Next, consider that $a$ is odd. Factoring the LHS gives $x^a - 1 = left(x - 1right)left(x^a-1 + x^a-2 + cdots + x + 1right)$. Since $x$ is odd and there are $a$ terms in the second factor, this factor must be odd and $gt 1$. However, the LHS of eqrefeq1 is a power of $2$, so it has no odd factors. Thus, $a$ can't be an odd integer.



            Putting this solution together with what you've shown proves that, apart from the one known solution, there are no other ones where $x$ or $y$ is a power of $2$.



            You could try to do something similar with odd prime numbers, but I don't believe that just these sorts of basic remainder and number of factors type arguments are sufficient to show there is no solution if either $x$ or $y$ is of a given simple form (e.g., just a prime or a power of a prime). If you haven't already done so, you may wish to read & analyze Preda Mihăilescu's proof, such as discussed & outlined in the AMS article at Catalan's Conjecture: Another Old Diophantine Problem Solved.






            share|cite|improve this answer









            $endgroup$

















              1












              $begingroup$

              I couldn't find any errors so your approach seems to be correct.



              Note, though, that your solution is more general in that it also works for any $x$ being a positive power of $2$, plus for all odd $y gt 1$, not just odd primes. For the first case, this is because your proof just requires the LHS to be a power of $2$. For the second case, this is due to you only using that $y$ is odd, and in particular not using any particular property of primes, in your solution.



              As for "exploring further" with this approach, I'm not quite sure what you mean. However, note you can use a similar approach to show that the only values which work for $y$ being a positive power of $2$ is the only known solution. To show this, you can follow a similar approach to what you did. First, let $y = 2^c$ for an integer $c ge 1$. Next, the equation can now be rewritten as



              $$2^bc = x^a - 1 tag1labeleq1$$



              This shows that $x$ must be odd. First, consider that $a$ is even, i.e., $a = 2d$. Then $x^a - 1 = left(x^d + 1right)left(x^d - 1right)$, so both $x^d + 1$ and $x^d - 1$ must be powers of $2$. First, $x^d - 1 = 1 ; Rightarrow ; x^d = 2$, so it's not possible. Next, $x^d - 1 = 2 ; Rightarrow ; x^d = 3$, so $x = 3$ and $d = 1$, giving the $1$ known solution. No larger values of $x^d - 1$ as a power of $2$ have that $x^d + 1$ is also a power of $2$.



              Next, consider that $a$ is odd. Factoring the LHS gives $x^a - 1 = left(x - 1right)left(x^a-1 + x^a-2 + cdots + x + 1right)$. Since $x$ is odd and there are $a$ terms in the second factor, this factor must be odd and $gt 1$. However, the LHS of eqrefeq1 is a power of $2$, so it has no odd factors. Thus, $a$ can't be an odd integer.



              Putting this solution together with what you've shown proves that, apart from the one known solution, there are no other ones where $x$ or $y$ is a power of $2$.



              You could try to do something similar with odd prime numbers, but I don't believe that just these sorts of basic remainder and number of factors type arguments are sufficient to show there is no solution if either $x$ or $y$ is of a given simple form (e.g., just a prime or a power of a prime). If you haven't already done so, you may wish to read & analyze Preda Mihăilescu's proof, such as discussed & outlined in the AMS article at Catalan's Conjecture: Another Old Diophantine Problem Solved.






              share|cite|improve this answer









              $endgroup$















                1












                1








                1





                $begingroup$

                I couldn't find any errors so your approach seems to be correct.



                Note, though, that your solution is more general in that it also works for any $x$ being a positive power of $2$, plus for all odd $y gt 1$, not just odd primes. For the first case, this is because your proof just requires the LHS to be a power of $2$. For the second case, this is due to you only using that $y$ is odd, and in particular not using any particular property of primes, in your solution.



                As for "exploring further" with this approach, I'm not quite sure what you mean. However, note you can use a similar approach to show that the only values which work for $y$ being a positive power of $2$ is the only known solution. To show this, you can follow a similar approach to what you did. First, let $y = 2^c$ for an integer $c ge 1$. Next, the equation can now be rewritten as



                $$2^bc = x^a - 1 tag1labeleq1$$



                This shows that $x$ must be odd. First, consider that $a$ is even, i.e., $a = 2d$. Then $x^a - 1 = left(x^d + 1right)left(x^d - 1right)$, so both $x^d + 1$ and $x^d - 1$ must be powers of $2$. First, $x^d - 1 = 1 ; Rightarrow ; x^d = 2$, so it's not possible. Next, $x^d - 1 = 2 ; Rightarrow ; x^d = 3$, so $x = 3$ and $d = 1$, giving the $1$ known solution. No larger values of $x^d - 1$ as a power of $2$ have that $x^d + 1$ is also a power of $2$.



                Next, consider that $a$ is odd. Factoring the LHS gives $x^a - 1 = left(x - 1right)left(x^a-1 + x^a-2 + cdots + x + 1right)$. Since $x$ is odd and there are $a$ terms in the second factor, this factor must be odd and $gt 1$. However, the LHS of eqrefeq1 is a power of $2$, so it has no odd factors. Thus, $a$ can't be an odd integer.



                Putting this solution together with what you've shown proves that, apart from the one known solution, there are no other ones where $x$ or $y$ is a power of $2$.



                You could try to do something similar with odd prime numbers, but I don't believe that just these sorts of basic remainder and number of factors type arguments are sufficient to show there is no solution if either $x$ or $y$ is of a given simple form (e.g., just a prime or a power of a prime). If you haven't already done so, you may wish to read & analyze Preda Mihăilescu's proof, such as discussed & outlined in the AMS article at Catalan's Conjecture: Another Old Diophantine Problem Solved.






                share|cite|improve this answer









                $endgroup$



                I couldn't find any errors so your approach seems to be correct.



                Note, though, that your solution is more general in that it also works for any $x$ being a positive power of $2$, plus for all odd $y gt 1$, not just odd primes. For the first case, this is because your proof just requires the LHS to be a power of $2$. For the second case, this is due to you only using that $y$ is odd, and in particular not using any particular property of primes, in your solution.



                As for "exploring further" with this approach, I'm not quite sure what you mean. However, note you can use a similar approach to show that the only values which work for $y$ being a positive power of $2$ is the only known solution. To show this, you can follow a similar approach to what you did. First, let $y = 2^c$ for an integer $c ge 1$. Next, the equation can now be rewritten as



                $$2^bc = x^a - 1 tag1labeleq1$$



                This shows that $x$ must be odd. First, consider that $a$ is even, i.e., $a = 2d$. Then $x^a - 1 = left(x^d + 1right)left(x^d - 1right)$, so both $x^d + 1$ and $x^d - 1$ must be powers of $2$. First, $x^d - 1 = 1 ; Rightarrow ; x^d = 2$, so it's not possible. Next, $x^d - 1 = 2 ; Rightarrow ; x^d = 3$, so $x = 3$ and $d = 1$, giving the $1$ known solution. No larger values of $x^d - 1$ as a power of $2$ have that $x^d + 1$ is also a power of $2$.



                Next, consider that $a$ is odd. Factoring the LHS gives $x^a - 1 = left(x - 1right)left(x^a-1 + x^a-2 + cdots + x + 1right)$. Since $x$ is odd and there are $a$ terms in the second factor, this factor must be odd and $gt 1$. However, the LHS of eqrefeq1 is a power of $2$, so it has no odd factors. Thus, $a$ can't be an odd integer.



                Putting this solution together with what you've shown proves that, apart from the one known solution, there are no other ones where $x$ or $y$ is a power of $2$.



                You could try to do something similar with odd prime numbers, but I don't believe that just these sorts of basic remainder and number of factors type arguments are sufficient to show there is no solution if either $x$ or $y$ is of a given simple form (e.g., just a prime or a power of a prime). If you haven't already done so, you may wish to read & analyze Preda Mihăilescu's proof, such as discussed & outlined in the AMS article at Catalan's Conjecture: Another Old Diophantine Problem Solved.







                share|cite|improve this answer












                share|cite|improve this answer



                share|cite|improve this answer










                answered Mar 29 at 1:58









                John OmielanJohn Omielan

                4,6162215




                4,6162215



























                    draft saved

                    draft discarded
















































                    Thanks for contributing an answer to Mathematics Stack Exchange!


                    • Please be sure to answer the question. Provide details and share your research!

                    But avoid


                    • Asking for help, clarification, or responding to other answers.

                    • Making statements based on opinion; back them up with references or personal experience.

                    Use MathJax to format equations. MathJax reference.


                    To learn more, see our tips on writing great answers.




                    draft saved


                    draft discarded














                    StackExchange.ready(
                    function ()
                    StackExchange.openid.initPostLogin('.new-post-login', 'https%3a%2f%2fmath.stackexchange.com%2fquestions%2f2153827%2fpartial-simplified-proof-for-the-prime-version-of-the-catalan-conjecture%23new-answer', 'question_page');

                    );

                    Post as a guest















                    Required, but never shown





















































                    Required, but never shown














                    Required, but never shown












                    Required, but never shown







                    Required, but never shown

































                    Required, but never shown














                    Required, but never shown












                    Required, but never shown







                    Required, but never shown







                    Popular posts from this blog

                    Triangular numbers and gcdProving sum of a set is $0 pmod n$ if $n$ is odd, or $fracn2 pmod n$ if $n$ is even?Is greatest common divisor of two numbers really their smallest linear combination?GCD, LCM RelationshipProve a set of nonnegative integers with greatest common divisor 1 and closed under addition has all but finite many nonnegative integers.all pairs of a and b in an equation containing gcdTriangular Numbers Modulo $k$ - Hit All Values?Understanding the Existence and Uniqueness of the GCDGCD and LCM with logical symbolsThe greatest common divisor of two positive integers less than 100 is equal to 3. Their least common multiple is twelve times one of the integers.Suppose that for all integers $x$, $x|a$ and $x|b$ if and only if $x|c$. Then $c = gcd(a,b)$Which is the gcd of 2 numbers which are multiplied and the result is 600000?

                    Ingelân Ynhâld Etymology | Geografy | Skiednis | Polityk en bestjoer | Ekonomy | Demografy | Kultuer | Klimaat | Sjoch ek | Keppelings om utens | Boarnen, noaten en referinsjes Navigaasjemenuwww.gov.ukOffisjele webside fan it regear fan it Feriene KeninkrykOffisjele webside fan it Britske FerkearsburoNederlânsktalige ynformaasje fan it Britske FerkearsburoOffisjele webside fan English Heritage, de organisaasje dy't him ynset foar it behâld fan it Ingelske kultuergoedYnwennertallen fan alle Britske stêden út 'e folkstelling fan 2011Notes en References, op dizze sideEngland

                    Հադիս Բովանդակություն Անվանում և նշանակություն | Դասակարգում | Աղբյուրներ | Նավարկման ցանկ